A integral about expotential function

  • Thread starter Thread starter athrun200
  • Start date Start date
  • Tags Tags
    Function Integral
Click For Summary
The discussion focuses on calculating the integral of the exponential function, specifically the Gaussian integral. A suggested method involves squaring the integral and converting from Cartesian to polar coordinates. This approach leads to the expression involving double integrals, which simplifies the calculation. There is a mention of the importance of understanding Gaussian integrals before studying quantum mechanics. The conversation emphasizes the necessity of foundational math skills for advanced topics in physics.
athrun200
Messages
275
Reaction score
0

Homework Statement



attachment.php?attachmentid=36742&stc=1&d=1309100124.jpg


Homework Equations


Unknown

The Attempt at a Solution



attachment.php?attachmentid=36743&stc=1&d=1309100124.jpg


How to calculate this integral?
I have tried substitution and by parts. But they fail to get the answer.
 

Attachments

  • 1.jpg
    1.jpg
    8.2 KB · Views: 455
  • 2.jpg
    2.jpg
    11.5 KB · Views: 484
Physics news on Phys.org
There is a trick for this, square the integral:
<br /> \left(\int_{-\infty}^{\infty}e^{-x^{2}}dx\right)^{2}=\int_{-\infty}^{\infty}\int_{-\infty}^{\infty}e^{-(x^{2}+y^{2})}dxdy<br />
Now the idea is to make a change of co-ordinates from cartesian to polar...
 
Are you taking quantum mechanics before knowing what a gaussian integral is ? Because you shouldn't...
 
dextercioby said:
Are you taking quantum mechanics before knowing what a gaussian integral is ? Because you shouldn't...

Oh you are right.
In fact, I am self-learning quantum mechanics, and I don't know what maths skills I need to have.
 
So move on from my explanation:
<br /> \int_{-\infty}^{\infty}\int_{-\infty}^{\infty}e^{-(x^{2}+y^{2})}dxdy=\int_{0}^{2\pi}\int_{0}^{\infty}re^{-r^{2}}drd\theta =2\pi\int_{0}^{\infty}re^{-r^{2}}dr<br />
I think I will leave the last bit to you as it's standard integration.
 
Question: A clock's minute hand has length 4 and its hour hand has length 3. What is the distance between the tips at the moment when it is increasing most rapidly?(Putnam Exam Question) Answer: Making assumption that both the hands moves at constant angular velocities, the answer is ## \sqrt{7} .## But don't you think this assumption is somewhat doubtful and wrong?

Similar threads

  • · Replies 22 ·
Replies
22
Views
2K
  • · Replies 5 ·
Replies
5
Views
2K
  • · Replies 4 ·
Replies
4
Views
2K
Replies
3
Views
805
  • · Replies 18 ·
Replies
18
Views
3K
  • · Replies 11 ·
Replies
11
Views
3K
  • · Replies 44 ·
2
Replies
44
Views
6K
  • · Replies 10 ·
Replies
10
Views
1K
  • · Replies 12 ·
Replies
12
Views
2K
  • · Replies 3 ·
Replies
3
Views
2K